Trả lời bởi giáo viên

Đáp án đúng: c

Mặt cầu tâm \(I\left( { - 1;2; - 3} \right)\), bán kính \(R = 2\sqrt 2 \) có phương trình \({\left( {x + 1} \right)^2} + {\left( {y - 2} \right)^2} + {\left( {z + 3} \right)^2} = 8\).

Hướng dẫn giải:

Mặt cầu tâm \(I\left( {{x_0};{y_0};{z_0}} \right)\) và bán kính \(R\) có phương trình \({\left( {x - {x_0}} \right)^2} + {\left( {y - {y_0}} \right)^2} + {\left( {z - {z_0}} \right)^2} = {R^2}\) 

Câu hỏi khác